Added topic tags to some S&J-4 problems + minor typos
[course.git] / latex / problems / Serway_and_Jewett_4 / problem28.56.tex
1 \begin{problem*}{28.56} % photoelectric effect
2 Figure P28.56 shows the stopping potential versus the incident photon
3 frequency for the photoelectric effect for sodium.  Use the graph to
4 find \Part{a} the work function, \Part{b} the ratio $h/e$,
5 and \Part{c} the cutoff wavelength.  The data are taken from
6 R.A.~Millikan, \emph{Physical Review} 7:362 (1916).
7 \begin{center}
8 \begin{asy}
9 import graph;
10 size(5cm, 4cm, IgnoreAspect);
11
12 real fmin = 400;
13 real fmax = 1200;
14
15 pair[] points={(550,0.827),(689,1.20),(741,1.39),
16                (818,1.79),(959,2.29),(1182,3.20)}; // (THz, V)
17
18 real fit(real f)
19 {
20   real work_fn = 1.62; // electron-volts
21   real h = 6.56e-34; // joule-seconds
22   real E = h*f*1e12 / 1.6e-19; // electron-volts
23   real Vs = E-work_fn;
24   return Vs;
25 }
26
27 draw(graph(fit, fmin, fmax), red);
28 draw(graph(points), p=invisible, marker=marker(scale(1pt)*unitcircle, blue));
29
30
31 pen thin=linewidth(0.5*linewidth())+grey;
32 xaxis("$f$ (THz)",BottomTop,
33       LeftTicks(begin=false,end=false,extend=true,ptick=thin));
34 yaxis("$\Delta V_s$ (V)",LeftRight,
35       RightTicks(begin=false,end=false,extend=true,ptick=thin));
36 \end{asy}
37 \end{center}
38 \end{problem*} % problem 28.56
39
40 \begin{solution}
41 \Part{a}
42 The fit line passes nearby the points $(400\U{THz},0\U{V})$ and
43 $(1.20\U{PHz},3.3\U{V})$.  In point-slope form, the fit line is then
44 \begin{align}
45   \Delta V_s-0\U{V} &= \frac{(3.3-0)\U{V}}{(1200-400)\U{THz}} (f-400\U{THz}) \\
46   \Delta V_s &= 4.125\text{mV/THz}\cdot (f-400\U{THz})
47 \end{align}
48 The work function is the inverse $y$-intercept, so
49 \begin{equation}
50   \phi = -\Delta V_s(f=0) = -4.125\text{mV/THz}\cdot(-400\U{THz})
51     = \ans{1.65\U{eV}}
52 \end{equation}
53
54 \Part{b}
55 The theoretical form for the fit line is
56 \begin{align}
57   e\Delta V_s &= hf - \phi \\
58   \Delta V_s &= \frac{h}{e}f - \phi
59 \end{align}
60 so $\frac{h}{e} = \ans{4.12\U{mV/THz}} = \ans{4.12\U{pV/s}}$.
61
62 \Part{c}
63 The cutoff wavelength is given by the work function and conservation
64 of energy.
65 \begin{align}
66   hf_\text{cut} &= \frac{hc}{\lambda_\text{cut}} = \phi \\
67   \lambda_\text{cut} &= \frac{hc}{\phi} = \ans{751\U{nm}}
68 \end{align}
69 \end{solution}